Lebesgue Measurable via open set












1












$begingroup$


I am a noob at measure theory so please forgive me for being naive, I just need help, here is what I have albeit this is a very very standard result:



Let $m$ denote Lebesgue measure on $mathbb{R}$ and $E$ a measurable subset with FINITE measure.



Show that,



$forall$ $epsilon > 0$, $exists$ open set $U$ containing $E$ s.t.



$m(U backslash E)$ < $epsilon$ (**)



What is the standard way to go about it because my intuition is that since $E$ is already measurable, then (**) becomes



$m(U backslash E)$ $leq$ $epsilon$



then we assume further that $E$ has FINITE measure,



so $E$ is somewhat "small"er than if it has infinite measure, but how does this guarantee that the open set containing $E$ has a "smaller difference" to $E$ which in turn gives us $<$ and not $leq$.



OR is there a subtlety on the fact that $m$ is Lebesgue measure and my problem only states $E$ is measurable, but when it states finite measure it says $m(E) < infty$ which assumes $E$ is Lebesgue Measurable? so This thought can't be correct.



I am using Stein Shakarchi mainly although I have papa Rudin for reference.



Any helps is greatly appreciated, if theres ANYTHING I said incorrectly or am doing wrong, PLEASE LET ME KNOW!!










share|cite|improve this question











$endgroup$








  • 1




    $begingroup$
    Don't apologize for asking questions! I appreciate well-researched "noob" questions more than PSQs.
    $endgroup$
    – Don Thousand
    Dec 18 '18 at 20:20






  • 1




    $begingroup$
    $<$ and $leq$ in this situation are exactly equivalent, since $epsilon$ is arbitrary and kept strictly positive. In any case, this result is called outer regularity, and exactly how you would like to prove it will depend somewhat on what you already know/assume about Lebesgue measurable sets. In particular, if you've defined the Lebesgue measure as the restriction of the Lebesgue outer measure to the Lebesgue measurable sets, then this result follows immediately from the definition of the Lebesgue outer measure.
    $endgroup$
    – Ian
    Dec 18 '18 at 20:24








  • 1




    $begingroup$
    (Cont.) By contrast, if, say, you define the Lebesgue measure by defining it on intervals and then applying an extension theorem to obtain the Lebesgue measure on the Borel $sigma$-algebra and then extend it by completion, then there is more work to be done. (By the way, the result is also true when $E$ has infinite measure.)
    $endgroup$
    – Ian
    Dec 18 '18 at 20:26












  • $begingroup$
    OMG you're right I cannot believe I missed that, the two inequalities are the same since we change by $epsilon$!! ok, so if the way I wrote up there IS how we defined a Lebesgue Measurable set Then like you say the finite case doesn't require much proof, its more definitions shoving around? that makes sense. And you're correct the result holds in general for any measurable set you can always find an open set small enough so my inequality holds. Another question I had was could you go about proving it using exterior measure and union of almost disjoint cubes?
    $endgroup$
    – Hossien Sahebjame
    Dec 18 '18 at 20:32






  • 1




    $begingroup$
    For any measurable $E,$ and $epsilon >0:$ For $nin Bbb N$ let $E_n=Ecap (-n,n)$ and let $U_n$ be open with $E_nsubset U_n$ and $m(U_n$ $E_n)<2^{-n}epsilon$. Let $U=cup_{nin Bbb N}U_n.$ Then $Usupset E$ and $U$ $E=$ $=(cup_{nin Bbb N}U_n) setminus E$ $=cup_{nin Bbb N}(U_n$ $E)subset$ $ cup_{nin Bbb N}(U_n$ $E_n).$ Therefore $m(U$ $E)leq$ $leq sum_{nin Bbb N}m(U_n setminus E_n)<epsilon.$
    $endgroup$
    – DanielWainfleet
    Dec 19 '18 at 3:58


















1












$begingroup$


I am a noob at measure theory so please forgive me for being naive, I just need help, here is what I have albeit this is a very very standard result:



Let $m$ denote Lebesgue measure on $mathbb{R}$ and $E$ a measurable subset with FINITE measure.



Show that,



$forall$ $epsilon > 0$, $exists$ open set $U$ containing $E$ s.t.



$m(U backslash E)$ < $epsilon$ (**)



What is the standard way to go about it because my intuition is that since $E$ is already measurable, then (**) becomes



$m(U backslash E)$ $leq$ $epsilon$



then we assume further that $E$ has FINITE measure,



so $E$ is somewhat "small"er than if it has infinite measure, but how does this guarantee that the open set containing $E$ has a "smaller difference" to $E$ which in turn gives us $<$ and not $leq$.



OR is there a subtlety on the fact that $m$ is Lebesgue measure and my problem only states $E$ is measurable, but when it states finite measure it says $m(E) < infty$ which assumes $E$ is Lebesgue Measurable? so This thought can't be correct.



I am using Stein Shakarchi mainly although I have papa Rudin for reference.



Any helps is greatly appreciated, if theres ANYTHING I said incorrectly or am doing wrong, PLEASE LET ME KNOW!!










share|cite|improve this question











$endgroup$








  • 1




    $begingroup$
    Don't apologize for asking questions! I appreciate well-researched "noob" questions more than PSQs.
    $endgroup$
    – Don Thousand
    Dec 18 '18 at 20:20






  • 1




    $begingroup$
    $<$ and $leq$ in this situation are exactly equivalent, since $epsilon$ is arbitrary and kept strictly positive. In any case, this result is called outer regularity, and exactly how you would like to prove it will depend somewhat on what you already know/assume about Lebesgue measurable sets. In particular, if you've defined the Lebesgue measure as the restriction of the Lebesgue outer measure to the Lebesgue measurable sets, then this result follows immediately from the definition of the Lebesgue outer measure.
    $endgroup$
    – Ian
    Dec 18 '18 at 20:24








  • 1




    $begingroup$
    (Cont.) By contrast, if, say, you define the Lebesgue measure by defining it on intervals and then applying an extension theorem to obtain the Lebesgue measure on the Borel $sigma$-algebra and then extend it by completion, then there is more work to be done. (By the way, the result is also true when $E$ has infinite measure.)
    $endgroup$
    – Ian
    Dec 18 '18 at 20:26












  • $begingroup$
    OMG you're right I cannot believe I missed that, the two inequalities are the same since we change by $epsilon$!! ok, so if the way I wrote up there IS how we defined a Lebesgue Measurable set Then like you say the finite case doesn't require much proof, its more definitions shoving around? that makes sense. And you're correct the result holds in general for any measurable set you can always find an open set small enough so my inequality holds. Another question I had was could you go about proving it using exterior measure and union of almost disjoint cubes?
    $endgroup$
    – Hossien Sahebjame
    Dec 18 '18 at 20:32






  • 1




    $begingroup$
    For any measurable $E,$ and $epsilon >0:$ For $nin Bbb N$ let $E_n=Ecap (-n,n)$ and let $U_n$ be open with $E_nsubset U_n$ and $m(U_n$ $E_n)<2^{-n}epsilon$. Let $U=cup_{nin Bbb N}U_n.$ Then $Usupset E$ and $U$ $E=$ $=(cup_{nin Bbb N}U_n) setminus E$ $=cup_{nin Bbb N}(U_n$ $E)subset$ $ cup_{nin Bbb N}(U_n$ $E_n).$ Therefore $m(U$ $E)leq$ $leq sum_{nin Bbb N}m(U_n setminus E_n)<epsilon.$
    $endgroup$
    – DanielWainfleet
    Dec 19 '18 at 3:58
















1












1








1





$begingroup$


I am a noob at measure theory so please forgive me for being naive, I just need help, here is what I have albeit this is a very very standard result:



Let $m$ denote Lebesgue measure on $mathbb{R}$ and $E$ a measurable subset with FINITE measure.



Show that,



$forall$ $epsilon > 0$, $exists$ open set $U$ containing $E$ s.t.



$m(U backslash E)$ < $epsilon$ (**)



What is the standard way to go about it because my intuition is that since $E$ is already measurable, then (**) becomes



$m(U backslash E)$ $leq$ $epsilon$



then we assume further that $E$ has FINITE measure,



so $E$ is somewhat "small"er than if it has infinite measure, but how does this guarantee that the open set containing $E$ has a "smaller difference" to $E$ which in turn gives us $<$ and not $leq$.



OR is there a subtlety on the fact that $m$ is Lebesgue measure and my problem only states $E$ is measurable, but when it states finite measure it says $m(E) < infty$ which assumes $E$ is Lebesgue Measurable? so This thought can't be correct.



I am using Stein Shakarchi mainly although I have papa Rudin for reference.



Any helps is greatly appreciated, if theres ANYTHING I said incorrectly or am doing wrong, PLEASE LET ME KNOW!!










share|cite|improve this question











$endgroup$




I am a noob at measure theory so please forgive me for being naive, I just need help, here is what I have albeit this is a very very standard result:



Let $m$ denote Lebesgue measure on $mathbb{R}$ and $E$ a measurable subset with FINITE measure.



Show that,



$forall$ $epsilon > 0$, $exists$ open set $U$ containing $E$ s.t.



$m(U backslash E)$ < $epsilon$ (**)



What is the standard way to go about it because my intuition is that since $E$ is already measurable, then (**) becomes



$m(U backslash E)$ $leq$ $epsilon$



then we assume further that $E$ has FINITE measure,



so $E$ is somewhat "small"er than if it has infinite measure, but how does this guarantee that the open set containing $E$ has a "smaller difference" to $E$ which in turn gives us $<$ and not $leq$.



OR is there a subtlety on the fact that $m$ is Lebesgue measure and my problem only states $E$ is measurable, but when it states finite measure it says $m(E) < infty$ which assumes $E$ is Lebesgue Measurable? so This thought can't be correct.



I am using Stein Shakarchi mainly although I have papa Rudin for reference.



Any helps is greatly appreciated, if theres ANYTHING I said incorrectly or am doing wrong, PLEASE LET ME KNOW!!







real-analysis measure-theory lebesgue-measure






share|cite|improve this question















share|cite|improve this question













share|cite|improve this question




share|cite|improve this question








edited Dec 18 '18 at 20:21







Hossien Sahebjame

















asked Dec 18 '18 at 20:19









Hossien SahebjameHossien Sahebjame

1049




1049








  • 1




    $begingroup$
    Don't apologize for asking questions! I appreciate well-researched "noob" questions more than PSQs.
    $endgroup$
    – Don Thousand
    Dec 18 '18 at 20:20






  • 1




    $begingroup$
    $<$ and $leq$ in this situation are exactly equivalent, since $epsilon$ is arbitrary and kept strictly positive. In any case, this result is called outer regularity, and exactly how you would like to prove it will depend somewhat on what you already know/assume about Lebesgue measurable sets. In particular, if you've defined the Lebesgue measure as the restriction of the Lebesgue outer measure to the Lebesgue measurable sets, then this result follows immediately from the definition of the Lebesgue outer measure.
    $endgroup$
    – Ian
    Dec 18 '18 at 20:24








  • 1




    $begingroup$
    (Cont.) By contrast, if, say, you define the Lebesgue measure by defining it on intervals and then applying an extension theorem to obtain the Lebesgue measure on the Borel $sigma$-algebra and then extend it by completion, then there is more work to be done. (By the way, the result is also true when $E$ has infinite measure.)
    $endgroup$
    – Ian
    Dec 18 '18 at 20:26












  • $begingroup$
    OMG you're right I cannot believe I missed that, the two inequalities are the same since we change by $epsilon$!! ok, so if the way I wrote up there IS how we defined a Lebesgue Measurable set Then like you say the finite case doesn't require much proof, its more definitions shoving around? that makes sense. And you're correct the result holds in general for any measurable set you can always find an open set small enough so my inequality holds. Another question I had was could you go about proving it using exterior measure and union of almost disjoint cubes?
    $endgroup$
    – Hossien Sahebjame
    Dec 18 '18 at 20:32






  • 1




    $begingroup$
    For any measurable $E,$ and $epsilon >0:$ For $nin Bbb N$ let $E_n=Ecap (-n,n)$ and let $U_n$ be open with $E_nsubset U_n$ and $m(U_n$ $E_n)<2^{-n}epsilon$. Let $U=cup_{nin Bbb N}U_n.$ Then $Usupset E$ and $U$ $E=$ $=(cup_{nin Bbb N}U_n) setminus E$ $=cup_{nin Bbb N}(U_n$ $E)subset$ $ cup_{nin Bbb N}(U_n$ $E_n).$ Therefore $m(U$ $E)leq$ $leq sum_{nin Bbb N}m(U_n setminus E_n)<epsilon.$
    $endgroup$
    – DanielWainfleet
    Dec 19 '18 at 3:58
















  • 1




    $begingroup$
    Don't apologize for asking questions! I appreciate well-researched "noob" questions more than PSQs.
    $endgroup$
    – Don Thousand
    Dec 18 '18 at 20:20






  • 1




    $begingroup$
    $<$ and $leq$ in this situation are exactly equivalent, since $epsilon$ is arbitrary and kept strictly positive. In any case, this result is called outer regularity, and exactly how you would like to prove it will depend somewhat on what you already know/assume about Lebesgue measurable sets. In particular, if you've defined the Lebesgue measure as the restriction of the Lebesgue outer measure to the Lebesgue measurable sets, then this result follows immediately from the definition of the Lebesgue outer measure.
    $endgroup$
    – Ian
    Dec 18 '18 at 20:24








  • 1




    $begingroup$
    (Cont.) By contrast, if, say, you define the Lebesgue measure by defining it on intervals and then applying an extension theorem to obtain the Lebesgue measure on the Borel $sigma$-algebra and then extend it by completion, then there is more work to be done. (By the way, the result is also true when $E$ has infinite measure.)
    $endgroup$
    – Ian
    Dec 18 '18 at 20:26












  • $begingroup$
    OMG you're right I cannot believe I missed that, the two inequalities are the same since we change by $epsilon$!! ok, so if the way I wrote up there IS how we defined a Lebesgue Measurable set Then like you say the finite case doesn't require much proof, its more definitions shoving around? that makes sense. And you're correct the result holds in general for any measurable set you can always find an open set small enough so my inequality holds. Another question I had was could you go about proving it using exterior measure and union of almost disjoint cubes?
    $endgroup$
    – Hossien Sahebjame
    Dec 18 '18 at 20:32






  • 1




    $begingroup$
    For any measurable $E,$ and $epsilon >0:$ For $nin Bbb N$ let $E_n=Ecap (-n,n)$ and let $U_n$ be open with $E_nsubset U_n$ and $m(U_n$ $E_n)<2^{-n}epsilon$. Let $U=cup_{nin Bbb N}U_n.$ Then $Usupset E$ and $U$ $E=$ $=(cup_{nin Bbb N}U_n) setminus E$ $=cup_{nin Bbb N}(U_n$ $E)subset$ $ cup_{nin Bbb N}(U_n$ $E_n).$ Therefore $m(U$ $E)leq$ $leq sum_{nin Bbb N}m(U_n setminus E_n)<epsilon.$
    $endgroup$
    – DanielWainfleet
    Dec 19 '18 at 3:58










1




1




$begingroup$
Don't apologize for asking questions! I appreciate well-researched "noob" questions more than PSQs.
$endgroup$
– Don Thousand
Dec 18 '18 at 20:20




$begingroup$
Don't apologize for asking questions! I appreciate well-researched "noob" questions more than PSQs.
$endgroup$
– Don Thousand
Dec 18 '18 at 20:20




1




1




$begingroup$
$<$ and $leq$ in this situation are exactly equivalent, since $epsilon$ is arbitrary and kept strictly positive. In any case, this result is called outer regularity, and exactly how you would like to prove it will depend somewhat on what you already know/assume about Lebesgue measurable sets. In particular, if you've defined the Lebesgue measure as the restriction of the Lebesgue outer measure to the Lebesgue measurable sets, then this result follows immediately from the definition of the Lebesgue outer measure.
$endgroup$
– Ian
Dec 18 '18 at 20:24






$begingroup$
$<$ and $leq$ in this situation are exactly equivalent, since $epsilon$ is arbitrary and kept strictly positive. In any case, this result is called outer regularity, and exactly how you would like to prove it will depend somewhat on what you already know/assume about Lebesgue measurable sets. In particular, if you've defined the Lebesgue measure as the restriction of the Lebesgue outer measure to the Lebesgue measurable sets, then this result follows immediately from the definition of the Lebesgue outer measure.
$endgroup$
– Ian
Dec 18 '18 at 20:24






1




1




$begingroup$
(Cont.) By contrast, if, say, you define the Lebesgue measure by defining it on intervals and then applying an extension theorem to obtain the Lebesgue measure on the Borel $sigma$-algebra and then extend it by completion, then there is more work to be done. (By the way, the result is also true when $E$ has infinite measure.)
$endgroup$
– Ian
Dec 18 '18 at 20:26






$begingroup$
(Cont.) By contrast, if, say, you define the Lebesgue measure by defining it on intervals and then applying an extension theorem to obtain the Lebesgue measure on the Borel $sigma$-algebra and then extend it by completion, then there is more work to be done. (By the way, the result is also true when $E$ has infinite measure.)
$endgroup$
– Ian
Dec 18 '18 at 20:26














$begingroup$
OMG you're right I cannot believe I missed that, the two inequalities are the same since we change by $epsilon$!! ok, so if the way I wrote up there IS how we defined a Lebesgue Measurable set Then like you say the finite case doesn't require much proof, its more definitions shoving around? that makes sense. And you're correct the result holds in general for any measurable set you can always find an open set small enough so my inequality holds. Another question I had was could you go about proving it using exterior measure and union of almost disjoint cubes?
$endgroup$
– Hossien Sahebjame
Dec 18 '18 at 20:32




$begingroup$
OMG you're right I cannot believe I missed that, the two inequalities are the same since we change by $epsilon$!! ok, so if the way I wrote up there IS how we defined a Lebesgue Measurable set Then like you say the finite case doesn't require much proof, its more definitions shoving around? that makes sense. And you're correct the result holds in general for any measurable set you can always find an open set small enough so my inequality holds. Another question I had was could you go about proving it using exterior measure and union of almost disjoint cubes?
$endgroup$
– Hossien Sahebjame
Dec 18 '18 at 20:32




1




1




$begingroup$
For any measurable $E,$ and $epsilon >0:$ For $nin Bbb N$ let $E_n=Ecap (-n,n)$ and let $U_n$ be open with $E_nsubset U_n$ and $m(U_n$ $E_n)<2^{-n}epsilon$. Let $U=cup_{nin Bbb N}U_n.$ Then $Usupset E$ and $U$ $E=$ $=(cup_{nin Bbb N}U_n) setminus E$ $=cup_{nin Bbb N}(U_n$ $E)subset$ $ cup_{nin Bbb N}(U_n$ $E_n).$ Therefore $m(U$ $E)leq$ $leq sum_{nin Bbb N}m(U_n setminus E_n)<epsilon.$
$endgroup$
– DanielWainfleet
Dec 19 '18 at 3:58






$begingroup$
For any measurable $E,$ and $epsilon >0:$ For $nin Bbb N$ let $E_n=Ecap (-n,n)$ and let $U_n$ be open with $E_nsubset U_n$ and $m(U_n$ $E_n)<2^{-n}epsilon$. Let $U=cup_{nin Bbb N}U_n.$ Then $Usupset E$ and $U$ $E=$ $=(cup_{nin Bbb N}U_n) setminus E$ $=cup_{nin Bbb N}(U_n$ $E)subset$ $ cup_{nin Bbb N}(U_n$ $E_n).$ Therefore $m(U$ $E)leq$ $leq sum_{nin Bbb N}m(U_n setminus E_n)<epsilon.$
$endgroup$
– DanielWainfleet
Dec 19 '18 at 3:58












0






active

oldest

votes











Your Answer





StackExchange.ifUsing("editor", function () {
return StackExchange.using("mathjaxEditing", function () {
StackExchange.MarkdownEditor.creationCallbacks.add(function (editor, postfix) {
StackExchange.mathjaxEditing.prepareWmdForMathJax(editor, postfix, [["$", "$"], ["\\(","\\)"]]);
});
});
}, "mathjax-editing");

StackExchange.ready(function() {
var channelOptions = {
tags: "".split(" "),
id: "69"
};
initTagRenderer("".split(" "), "".split(" "), channelOptions);

StackExchange.using("externalEditor", function() {
// Have to fire editor after snippets, if snippets enabled
if (StackExchange.settings.snippets.snippetsEnabled) {
StackExchange.using("snippets", function() {
createEditor();
});
}
else {
createEditor();
}
});

function createEditor() {
StackExchange.prepareEditor({
heartbeatType: 'answer',
autoActivateHeartbeat: false,
convertImagesToLinks: true,
noModals: true,
showLowRepImageUploadWarning: true,
reputationToPostImages: 10,
bindNavPrevention: true,
postfix: "",
imageUploader: {
brandingHtml: "Powered by u003ca class="icon-imgur-white" href="https://imgur.com/"u003eu003c/au003e",
contentPolicyHtml: "User contributions licensed under u003ca href="https://creativecommons.org/licenses/by-sa/3.0/"u003ecc by-sa 3.0 with attribution requiredu003c/au003e u003ca href="https://stackoverflow.com/legal/content-policy"u003e(content policy)u003c/au003e",
allowUrls: true
},
noCode: true, onDemand: true,
discardSelector: ".discard-answer"
,immediatelyShowMarkdownHelp:true
});


}
});














draft saved

draft discarded


















StackExchange.ready(
function () {
StackExchange.openid.initPostLogin('.new-post-login', 'https%3a%2f%2fmath.stackexchange.com%2fquestions%2f3045648%2flebesgue-measurable-via-open-set%23new-answer', 'question_page');
}
);

Post as a guest















Required, but never shown

























0






active

oldest

votes








0






active

oldest

votes









active

oldest

votes






active

oldest

votes
















draft saved

draft discarded




















































Thanks for contributing an answer to Mathematics Stack Exchange!


  • Please be sure to answer the question. Provide details and share your research!

But avoid



  • Asking for help, clarification, or responding to other answers.

  • Making statements based on opinion; back them up with references or personal experience.


Use MathJax to format equations. MathJax reference.


To learn more, see our tips on writing great answers.




draft saved


draft discarded














StackExchange.ready(
function () {
StackExchange.openid.initPostLogin('.new-post-login', 'https%3a%2f%2fmath.stackexchange.com%2fquestions%2f3045648%2flebesgue-measurable-via-open-set%23new-answer', 'question_page');
}
);

Post as a guest















Required, but never shown





















































Required, but never shown














Required, but never shown












Required, but never shown







Required, but never shown

































Required, but never shown














Required, but never shown












Required, but never shown







Required, but never shown







Popular posts from this blog

To store a contact into the json file from server.js file using a class in NodeJS

Redirect URL with Chrome Remote Debugging Android Devices

Dieringhausen